Prove $\frac{1}{\sqrt{a+8b}}+\frac{1}{\sqrt{b+8c}}+\frac{1}{\sqrt{c+8a}}\ge 1$ for $ab + bc + ca = 3$

algebra-precalculusholder-inequalityinequalitysymmetric-polynomialsuvw

Let $a,b,c\ge 0: ab+bc+ca=3.$ Prove that$$\frac{1}{\sqrt{a+8b}}+\frac{1}{\sqrt{b+8c}}+\frac{1}{\sqrt{c+8a}}\ge 1.$$
This problem is from a book.

This cyclic inequality becomes an equality at $a=b=c=1.$ I try to used some estimate which based on this point $(1,1,1)$.

By AM-GM$$\sum_{cyc}\frac{1}{\sqrt{a+8b}}\ge 6\sum_{cyc}\frac{1}{a+8b+9}, $$but$$\frac{1}{a+8b+9}+\frac{1}{b+8c+9}+\frac{1}{c+8a+9}\ge \frac{1}{6}$$ is already wrong at $a=\dfrac{\sqrt{30}}{50},b=5\sqrt{30},c=0.$

The following Cauchy-Schwarz also doesn't help here since $$\sqrt{a+8b}+\sqrt{b+8c}+\sqrt{c+8a}\le 9,$$ is doesn't hold when $a=\dfrac{\sqrt{30}}{50},b=5\sqrt{30},c=0.$ Even $(a+8b)(b+8c)(c+8a)\le 27$ leads wrong inequality with same counter example.

Now, I tried Holder$$\left(\sum_{cyc}\frac{1}{\sqrt{a+8b}}\right)^2.\sum_{cyc}\left[(a+8b)(ma+nb+pc)^3\right]\ge \left[(m+n+p)(a+b+c)\right]^3,$$but it seems hard. I can not find any special $(m,n,p)$.

I think my Holder estimae is not good enough. Hope to see better ideas. Thank you.

Best Answer

The Contradiction method helps!

Indeed, let $\sum\limits_{cyc}\frac{1}{\sqrt{a+8b}}<1$,$a=kx$, $b=ky$ and $c=kz$ such that $k>0$ and $$\sum_{cyc}\frac{1}{\sqrt{x+8y}}=1.$$

Thus, $$\frac{1}{\sqrt{k}}\sum_{cyc}\frac{1}{\sqrt{x+8y}}<\sum_{cyc}\frac{1}{\sqrt{x+8y}},$$ which gives $k>1$ and $$3=ab+ac+bc=k^2(xy+xz+yz)>xy+xz+yz,$$ which is a contradiction because we'll prove now that $$xy+xz+yz\geq3$$ for any positives $x$, $y$ and $z$ such that $\sum\limits_{cyc}\frac{1}{\sqrt{x+8y}}=1.$

Now, let $\frac{1}{\sqrt{x+8y}}=\frac{p}{3},$ $\frac{1}{\sqrt{y+8z}}=\frac{q}{3}$ and $\frac{1}{\sqrt{z+8x}}=\frac{r}{3}.$

Thus, $$p+q+r=3,$$ $$x=\frac{\frac{1}{p^2}-\frac{8}{q^2}+\frac{64}{r^2}}{57}\geq0,$$ $$y=\frac{\frac{1}{q^2}-\frac{8}{r^2}+\frac{64}{p^2}}{57}\geq0$$ and $$z=\frac{\frac{1}{r^2}-\frac{8}{p^2}+\frac{64}{q^2}}{57}\geq0$$ and we need to prove that: $$\sum_{cyc}\left(\frac{1}{p^2}-\frac{8}{q^2}+\frac{64}{r^2}\right)\left(\frac{1}{q^2}-\frac{8}{r^2}+\frac{64}{p^2}\right)\geq3\cdot57^2$$ or $$\sum_{cyc}(65p^4q^2r^2-8p^4q^4)\geq171p^4q^4r^4,$$ which after substitution $p+q+r=3u$, $pq+pr+qr=3v^2$, $pqr=w^3$ gives $$65(9u^2-6v^2)u^4w^6-8(18u^2w^6+12v^2w^6-108uv^4w^3+81v^8)u^4\geq171w^{12}$$ or $f(w^3)\geq0,$ where $$f(w^3)=-19w^{12}+49u^6w^6-54u^2v^4w^6+96u^5v^4w^3-72u^4v^8.$$ But by Maclaurin: $$f'(w^3)=96u^5v^4+98u^6w^3-108u^2v^4w^3-76w^9>0,$$ which says that it's enough to prove $f(w^3)\geq0$ for the minimal value of $w^3$.

Now, $p$, $q$ and $r$ are positive roots of the equation: $$(t-p)(t-q)(t-r)=0$$ or $$t^3-3ut^2+3v^2t-w^3=0$$ or $g(t)=w^3$, where $$g(t)=t^3-3ut^2+3v^2t.$$

Let $u=constant$ and $v=constant$ and we want to move $w^3$.

During this moving should be that the equation $g(t)=w^3$ has three positive roots.

But $$g'(t)=3t^2-6ut+3v^2=3(t-t_1)(t-t_2),$$ where $t_1=u-\sqrt{u^2-v^2}$ and $t_2=u+\sqrt{u^2-v^2},$ which says $t_{max}=t_1$ and $t_{min}=t_2$.

Also, we have: $g(0)=0$ and we can draw a graph of $g$, which intersects with a line $y=w^3$ in three points or maybe, if this line is a tangent line to the graph of $g$, so they have two common points.

We see that $w^3$ gets a minimal value, when $y=w^3$ is a tangent line to a graph of $g$ in the point $(t_2,g(t_2))$. Also, we need to check, what happens for $w^3\rightarrow0^+$.

Id est, it's enough to prove $f(w^3)\geq0$ for equality case of two variables

(the case $w^3\rightarrow0^+$ is impossible because it should be $\sum\limits_{cyc}(65a^4b^2c^2-8a^4b^4)>0$).

Now, let $q=p$ and $r=3-2p$.

Thus, $$0<p<\frac{3}{2},$$ $$\frac{64}{(3-2p)^2}-\frac{7}{p^2}\geq0$$ and $$\frac{65}{p^2}-\frac{8}{(3-2p)^2}\geq0,$$ which gives $$\frac{4\sqrt7-7}{6}\leq p\leq\frac{65-\sqrt{130}}{42}$$ and we need to prove that $$130p^6(3-2p)^2+65p^4(3-2p)^4-8(p^8+2p^4(3-2p)^4)\geq171p^8(3-2p)^4$$ or $$(p-1)^2(441-294p+277p^2+152p^3-1368p^4+1216p^5-304p^6)\geq0,$$ which is true for $\frac{4\sqrt7-7}{6}\leq p\leq\frac{65-\sqrt{130}}{42}$.